« first day (1940 days earlier)      last day (3082 days later) » 
00:00 - 17:0017:00 - 00:00

12:00 AM
@Chris'ssistheartist would a trip integral be volume under a three dimensional surface?
 
@Michael aha, you don't seem a high school student at all.
 
user174558
@Chris'ssistheartist Maybe he is deceiving us. Maybe he is a professor, lol.
 
@Chris'ssistheartist haha
@JasperLoy Yes, this was all a test of your analytical skills. No on a serious note, I am actually a good highschool math student. No more
@Chris'ssistheartist but I am interested. So adding integralas adds dimensions?
 
user174558
@Michael Very strange your school has no HL math.
 
user174558
@Michael Difficult to answer your random questions like that.
 
12:07 AM
Interestingly: if you put a unit circle in the plane tangent to the vertical line at the origin, then every line through the origin can be identified with a point on the circle. Rotating the lines in the plane by an angle of θ corresponds to rotating the points on that circle by 2θ.
 
@JasperLoy My math right now is legimately a joke. We just finished principles of derivation. Not allowed to use l'hopsitals law. Derivation: not allowed to use any of the cool techniques I saw on this website
 
user174558
@Michael You may look up multiple integrals and iterated integrals and Fubini's theorem.
 
@Michael I think you already know the answer.
 
volume under a surface can be calculated with double integrals just as areas under curves are calculated with single integrals. a triple integral of density will give mass of a three-dimensional body though @Michael
 
@JasperLoy so can you calculate the change in value of switching from integrals? Kind of like differentiating.
 
user174558
12:09 AM
@Michael Difficult to answer your random questions.
 
user174558
@Michael I think you already have a lot of reading to do before asking any more.
 
@JasperLoy no please mr banana!
 
@anon He knows the answers already, but it's kind from you to tell it.
 
What?
 
user174558
@Michael Thanks for calling me banana, I love it.
 
12:11 AM
@Chris'ssistheartist ok I actually don't know the answer to this one. Since you can calculate the change in x in a graph, can you calculate the change of the value of the function as the number of integrals changes?
 
the first challenge at hand would be to articulate that in a way that can be understood
 
user174558
@Michael This is a very confusing statement that does not make sense.
 
@anon do you refer to me?
 
my "What?" refers to you, my "challenge" refers to Michael
 
user174558
12:13 AM
I think Michael is anon's sockpuppet account.
 
@JasperLoy So so, is there a relation between the changes in values after a function
@JasperLoy is applied by n integrals
 
user174558
@Michael Your question does not make sense at all. So this means it is time to do more reading and less asking.
 
@JasperLoy Yes mr Banana. So what should I read about?
 
user174558
@Michael Anything you want, I have already given you suggestions above...
 
@JasperLoy was it pure mathematics?
 
user174558
12:15 AM
@Michael That too, and Wikipedia pages.
 
@JasperLoy what is a curl in math?
 
user174558
@Michael It is good to have questions, but too many questions without knowing anything is useless. In the first place, the questions do not make sense, or at least you are not describing it properly
 
user174558
@Michael You can find all the answers online. No more questions today. =)
 
@JasperLoy Oh ok. Thank you so much for dealing with my nonesense! Have a great day
 
user174558
@Michael When you become a doctor, try to cure as many as you can.
 
12:18 AM
@JasperLoy I will I promise. I promise I will try to help as many people as possible so there can be more mathematicians like you to help idiots like me!
 
user174558
@Michael I am not a mathematician. I am a mental patient.
 
user174558
Hello @ted.
 
@anon: Thanks for answering the question, but I don't follow it.
Hi @Jasper.
 
guys, pls help me out with a question
Let $X=Ult(\mathbb{N})$ be the set of all ultrafilters on $\mathbb{N}$, and consider the topological space $(Ult(\mathbb{N}),\tau)$ with $\tau$ being a topology with a basis $
\left \{ \hat{M}: M \subset \mathbb{N} \right \}$, where $\hat{M}=\left \{ U: U \in Ult(\mathbb{N}), M \in U \right \}$.
 
Hi @TedShifrin and @JasperLoy
 
user174558
12:20 AM
@morphic Bart, Sajindia is Twink but maybe I am wrong
 
@Michael: I did not allow students to use L'Hôpital early on in calculus, either. For example, when you use L'Hôpital's rule to do the limit $\lim\limits_{x\to 0}\frac{\sin x}x$, you are assuming the limit you're trying to do :)
 
@JasperLoy Why did he make a new account
 
the message is all weirdly formatted here in chat, but it displays fine in the preview where I wrote it (on the main site)
 
@Jasper: I think you're wrong. Twink is in Montreal, so I would be very surprised if he spoke Spanish rather than French. But maybe ...
 
anybody have an idea what's wrong?
 
user174558
12:22 AM
@TedShifrin How do you know he is in Montreal? You met him?
 
Let $f:Y \rightarrow \mathbb{R}$ be a bounded continuous function, where $Y$ is a subset of $X$, which contains only the principal ultrafilters - $\left \{A: A \subset \mathbb{N}, \left \{n \right \} \subset A \right \} $ for some $n \in \mathbb{N}$.

Show that this function can be continuously extended to $X$.
 
Jake, I've never played with ultrafilters in my entire life, and I'm not going to start now.
 
I guess it looks normal now.
 
No, haven't met him, but there were discussions about the geometry course he was taking. Maybe it was all a farce, but I dunno.
 
Don't you usually cover ultrafilters in introductory point-set topology
 
12:22 AM
:) no problem
 
NOOOOOO @morphic.
 
user174558
@morphic No.
 
Mr eyeglasses, you have the strangest ideas of what is "basic" or "elementary" based on your school curriculum!
 
user174558
@morphic The sections on filters in topology books are usually starred.
 
@Michael your try is pretty funny in a way, but I assure you that for my questions it might not be enough a life to answer them, with PhD or not.
@Michael Let's take one
Calculate by series manipulation only $$\sum_{n=1}^{\infty} \left(\frac{H_n}{n}\right)^3$$
 
12:24 AM
Not even in Munkres, and I recall Kelley's book covered nets, but I don't remember ultrafilters.
I no longer have all the books, so I can't look.
 
@Michael you're free to use any resources existing on earth. I mean I have nothign against if you call another 1000 PhD people like you and thing together of my question.
 
user174558
@TedShifrin Will you be celebrating Thanksgiving with family?
 
Nope, haven't done that in about 35 years, @Jasper.
 
user174558
@TedShifrin Oh, maybe you should, this Christmas or something.
 
I went to a Thanksgiving party at my school and that will be the extent of my celebration
 
12:26 AM
I decided long ago that cooking for good friends was a lot more fun than dealing with family, although my sister and her husband and I get along great.
For most of the last 30 years, I invited friends and lost souls to my house and spent 3 days cooking.
 
@Michael calculate in closed form by real methods $$\int_{-1}^1 \frac{\log^2(1-x)}{\pi^2+(2 \operatorname{arctanh}(x))^2} \, dx$$ in the next 10 years.
 
Several TV show scenes are being filmed in some our buildings this week so it's kind of crowded here
 
user174558
@TedShifrin Aha. I hope your mum is OK.
 
She hasn't known who I am in about 4-5 years, @Jasper. I will visit her in the nursing home when I go back to ATL in January.
 
@Michael let me give you one to think of for 20 years.
 
12:28 AM
:(
 
@Chris'ssistheartist Oh. My. God. My brain hurts. Ok I will clip mark that and I'll be back in a quick 10.
 
user174558
@morphic Maybe they are looking for you Mr Handsome.
 
wow, @morphic, what TV show?
 
@TedShifrin Three of them: Elementary, The Blacklist, and Blindspot. I have seen them filming Blindspot near my church before a couple months ago
 
Wow, I'll have to do serious googling.
 
12:29 AM
@TedShifrin They are young people TV shows, I think
 
LOL ... you mean younger than you?
 
They said there will be fake gunfire noises on campus lol
 
Maybe they make money on campus this way.
 
Well Elementary is about a modern version of Sherlock Holmes. It has Jonny Lee Miller (he was the main character in the movie Hackers but I don't know what else he's been in), and also Lucy Liu is in it
 
user174558
I want to join the show Mr Bachelor, lol.
 
12:31 AM
$$\sum_{n=1}^{\infty} \left(\left(\zeta(2)-1-\frac{1}{ 2^2}-\cdots -\frac{1}{n^2}\right)} \left(\zeta(3)-1- \frac{ 1}{2^3}-\cdots -\frac{1}{n^3} \right)\left(\zeta(4)-1-\frac{1}{2^4}-\cdots - \frac{1}{n^4}\right)\right)^2$$
 
Blindspot is based on a book series of the same title I think. It has the main character from the new 300 movie and the lady in the Thor movies that is in Thor's gang
 
that doesn't sound like young persons' stuff, mr eyeglasses. And Blacklist is FBI drama. Although I used to love James Spader.
 
Oh, I only know James Spader from The Office (US version) lol
Too bad I didn't get any photos today. Too busy with schoolwork
 
user174558
I love The Wonder Years!!!
 
I remember Spader from movies like Sex, Lies, Videotape, plus he was in Boston Legal for years.
 
12:33 AM
Oh, I've never heard of any of those
 
Is there a way I can show someone an equation I used in word and they can check if it is right?
 
@Michael $$\int _0^{\infty }\int _0^{\infty }\int _0^{\infty }\frac{\left(x \cot \left(\frac{x}{2}\right)-2\right) \left(y \cot \left(\frac{y}{2}\right)-2\right) \left(z \cot \left(\frac{z}{2}\right)-2\right) e^{-x y z}}{xyz}dx dy dz$$
 
We hate Word here. :)
 
user174558
But we love words. =)
 
One movie, morphic, Sex, Lies, and Videotape, I think.
 
12:34 AM
@Michael come here with all you best people you know and give me a lesson in my area
 
@TedShifrin Do you cringe whenever a math paper is written in Word
 
another one
$$\sum_{n=1}^{\infty} \left(\left(\zeta(2)-1-\frac{1}{ 2^2}-\cdots -\frac{1}{n^2}\right) \left(\zeta(3)-1- \frac{ 1}{2^3}-\cdots -\frac{1}{n^3} \right)\left(\zeta(4)-1-\frac{1}{2^4}-\cdots - \frac{1}{n^4}\right)\right)^2$$
 
No one does it any more, seriously, morphic.
 
@Chris'ssistheartist is that a challenge to the god of highschool integrals himself?
 
Some old fuddy duddies type homework assignments in Word.
 
12:35 AM
How can I upload an image?
 
Hit "upload," @Michael.
 
where is it good sir?
 
Next to "Send" to the right of the text box you type in.
 
user174558
@Michael The bottom.
 
@Michael Look, I just changed my mind: show me a brilliant idea, not a whole solution and I count it as if you had a whole solution.
 
12:36 AM
@JasperLoy do you need certain reputation to use it?
 
When a student was submitting math homework via e-mail here, the professor asked him to attach it as a .doc file lol...
 
user174558
@Michael I don't know. Ask the papaya.
 
@morphic: Here you go.
 
Wow, 1989
The only movie I know from that year is Batman
 
LOL, I was already tenured by then.
 
12:36 AM
Wow!!!
 
user174558
I love all the 4 Indiana Jones movies.
 
@Michael I don't know what stuff you created solved so far, but you talk to someone that created thousands of problems, not baby questions.
@Michael but it would be so great to give me your brilliant solutions! To learn from you or from your other 1000 friends of yours with PhD.
 
Harrison Ford kind of looks like he'd be a grumpy guy in real life
 
user174558
@Chris'ssistheartist I remember I called Sayan a baby and he was upset, ROFL.
 
@JasperLoy Who is Sayan
 
user174558
12:38 AM
@morphic I bet he can run faster than me.
 
@Michael I come in a TV show with you all, I mean to make a nice challenge with much fun.
 
@Chris'ssistheartist I just solved the twin prime conjecture.
 
user174558
@morphic Remember Me
 
@JasperLoy I haven't seen him in a while
 
user174558
@Michael Are you a troll?
 
12:39 AM
@JasperLoy it was a joke
 
LOL, you shouldn't ask such questions, @Jasper, since you troll with the best of 'em.
 
@Michael If you wanna challenge me, then let's do it. So, tell me a word about any of the stuff above. What does it go better, tan or hyperbolic functions?
 
user174558
=)
 
@Chris'ssistheartist easy. you see, there is a. there is a... There is a 2^2. Aha. I saw that. Checkmate. Hyperbolic
 
user174558
@Chris'ssistheartist I don't think he is able to answer you.
 
12:40 AM
@Michael What would you do in front of this stuff, really, other than admire it profoundly? It's the only thing you can do, to admire it much and hope one day I might like to show you some ways to calculate them.
 
user174558
Wait, is Sajindia male or female? I cannot tell from the pic @TedShifrin and @morphic.
 
@Michael but I'm not sure I wanna show you anything since you seem to know so much stuff.
 
@Chris'ssistheartist well first I need to get this chat to look like normal and not dollar signs. Then maybe give it 20 years. THen I might try to understand a part of it.
 
May identify as female, @Jasper, but cis male, as far as I can tell.
 
user174558
@Michael You might take 20 years to get this chat to look normal, LOL.
 
12:42 AM
@Chris'ssistheartist imgur.com/CZLc35C Will this function work for arc length of a quadratic?
 
my god, I hate it when I get stuck on a problem and can't let it go.
 
@JasperLoy I think male; I saw he made a video about coming out as gay or something
 
I'm starting to think my OCD tendencies and math don't go together very well.
 
user174558
@Sajindia Are you the user called Twink?
 
user174558
@Jake1234 Sorry to hear. I know it is hell.
 
12:43 AM
imgur.com/CZLc35C . WOuld this function be a good way to calculate the arc length of a function between two points?
Excluding the fact that it is not definite
I can always make it a definite
 
I don't see why all this meddling into identities is needed, @Jasper.
 
user174558
@TedShifrin OK. It's just so that at least we know who is who.
 
@Michael it seems you don't like that much what I call the real stuff.
 
@JasperLoy likes to be a detective
 
We certainly have no idea who chris'ssis is. Leave it alone.
 
user174558
12:45 AM
Hey @Sajindia if you want to talk, let me know.
 
@Chris'ssistheartist Haha I guess. No but can you seriously check this equation imgur.com/CZLc35C . I have an assigment due tomorrow that requires me to use somethign similar
 
@Michael I also have an assignment, amongts many other things, not to pay attention to these challenges and see of my work. You said you know nothing about integration, and then you put questions that come from a different level and try to laugh.
 
user174558
@Michael Does not mean anything when you have not specified your variables...
 
find the length between 45 and 1200
its an indefinite
 
user174558
@Michael You seem to jump from question to question. That is not good.
 
12:47 AM
My mistake I entered this conversation.
 
@JasperLoy Ok this is actually dead serious. This is something my level.
 
user174558
@Chris'ssistheartist You can leave it anytime.
 
user174558
@Michael But you cannot ask the expert such an elementary question. If you are serious, post on the main site. The expert is tired.
 
@JasperLoy but the expert likes helping bad people right, mrs expert?
@Chris'ssistheartist Please say yes
 
user174558
@Michael No, you seem like a troll now.
 
12:49 AM
@JasperLoy I SWEAR TO GOD THIS IS AN EQUATION THAT I DERIVED. I need to make sure it can calculate the length of a quadratic in the form of y=ax^2+bx+c
 
user174558
@Michael OK, post the question properly written on the main site if you are so serious.
 
I did
 
user174558
Then wait, LOL. End of story.
 
0
Q: Need help with a certain integral

MichaelSorry, this is my first post ever and formatting is bad. I appreciate all assistance Given the equation y= $ 3.8x^2-4.4x+1444 $ Find the definite arc length integral between 0 and 1200. I am not sure on how to format $(dy/dx)^2$ ,so I left it alone. $$\int_0^{1200} \sqrt{1+ (\frac{dy}{dx}}...

I got an answer, I just need to make sure I understand it and it is correct
I don't know why everyone is being mean. I am asking for help in understanding
Nvm. I'll go fail
 
user174558
@morphic Not as much of a detective as @alexclark.
 
12:54 AM
@Michael I think you only try (unsuccessfully) to make fun of people since some minutes ago you seemed very in trouble with simple differentiations. In the meantime you finished some other classes.
 
@Chris'ssistheartist I wasn't making fun of anyone. I was sarcastically saying I had a phd...
@Chris'ssistheartist I thought we were going on with the sarcasm.
@Chris'ssistheartist but I can't force you to help... I'll pretend it is right and just go with it.
 
@TedShifrin I put it over here
1
Q: Finitely-presented torsionfree group whose abelianization has torsion

whackaIn chat someone asked Does anyone know of a torsion-free group (finitely presentable) whose abelianisation has torsion?

spose I should ping @DanRust too
 
Lagange :D
hi @anon
 
hello
 
@Michael How would you calculate $$\int_0^1 x^{n-1} \log^3(1-x) \ dx$$? In case you're interested ... (I think this one shouldn't be a difficulty for you)
 
12:59 AM
I can't see it, sorry
 
hi @TedShifrin
 
@Michael It's what I call to be a light integral. You can finish it even mentally, seriously while jogging, doing some shopping and other activities.
 
@Chris'ssistheartist Uhmm, sec let me try to get this chat thing to work.
 
@TedShifrin do you know about phase space and configuration space ?
 
@Chris'ssistheartist Wait so how far does your knowledge of integrals go?
 
user174558
1:01 AM
@Michael I did not call her world expert for nothing.
 
I'm out to take some sleep.
 
user174558
@Michael I think you should not waste her time anymore.
 
user174558
@Chris'ssistheartist Goodnight.
 
@JasperLoy Good night.
 
user174558
Happy Thanksgiving Day to America! Good night!
 
1:09 AM
Is every covering space $p : E \to X$ induced by a properly discontinuous action on $E$ of a group $G$ so that $X=E/G$?
 
those are the regular/normal/Galois covers aren't they?
 
I don't know
so not every covering space has that property?
 
 
1 hour later…
2:32 AM
@anon Didn't this get answered in chat as well...
Well I upgraded my comment in chat to an answer there..
 
3:06 AM
hello
Can you have an n'th integral?
 
3:25 AM
nvm
 
 
2 hours later…
5:46 AM
Hi friends
How are we all?
Hello @TobiasKildetoft. Are you a student?
Don't know if people usually ask that sort of question here
 
@I'mmostlyjustanidiot I am a postdoc
 
Oh wow, and it's meant to be really hard to get a postdoc position I have heard, something like 1% of people get one or something
 
@I'mmostlyjustanidiot They are certainly competitive, but I think it is way more than 1%, at least if you only count those who actually apply for them
 
Oh, well that's very good to hear then. I am not a very good student because I'm not very smart, so I am trying to work extra hard to succeed
I'm taking a gap year now, so I have an extra year of preparation for my next year of university
Question: Why do we bother to name idempotent elements?
 
6:05 AM
@I'mmostlyjustanidiot Because it is an important property
they are for example related to the decomposition of rings into direct products
 
So the book lets $x\in A$ have $x^2=x$, then says $x'=1-x$ which clearly is also idempotent, since $(x')^2=(1-x)^2=1-2x+x^2=1-2x+x=1-x=x'$. We obtain $x+x'=1$ and $x-x^2=0$.

They then state that $x$ and $x'$ are 'complementary orthogonal idempotents', does that literally just mean that $x+x'=1$ and $xx'=0$ where $x,x'$ are idempotents?
Then they say that with this, we can take $a=ax+ax'$ for any $a\in A$ and we have that $A$ is a direct sum of rings $A=A_1\oplus A_2$ where $A_1=Ax$ and $A_2=Ax'$
 
@I'mmostlyjustanidiot Yes, that is precisely what complementary means here
 
That's a beautiful construction
 
 
1 hour later…
7:32 AM
Why does complex Lie algebra $g$ being solvable imply that $g'$ is nilpotent?
I can show the converse easily
 
7:53 AM
@robjohn can you explain how $$\binom{-\frac12}{k}=\left(-\frac14\right)^k\binom{2k}{k}$$? Is this a result of a formula?
 
8:27 AM
@I'mmostlyjustanidiot Yeah, the revere direction is a lot more tricky
It is because a solvable Lie algebra is isomorphic to a subalgebra of the algebra of upper triangular matrices
 
@TobiasKildetoft In the finite case I got an awesome answer from someone
 
(usually formulated in terms of stabilizing a full flag)
 
@TobiasKildetoft Yep that's exactly what he went for: math.stackexchange.com/a/1547042/284674
 
@I'mmostlyjustanidiot Yep, that's the result (and I agree that it probably fails for infinite dimensional Lie algebras)
though it might still work if one assumes it to be locally finite or something like that
 
What does locally finite mean, if it isn't too complicated
 
8:30 AM
@I'mmostlyjustanidiot It means that any finitely generated subalgebra is finite dimensional
 
8:57 AM
is there some inequality relating (x - y)^2 to x^2 + y^2? Or is there nothing we can say about the relationship between these two expressions?
 
@Riggs Their difference is precisely $-2xy$ (or $2xy$ depending on the order)
so it is determined by the signs of $x$ and $y$
 
9:13 AM
Can anyone have a look at this? I'm not sure if I'm doing this correctly and am not sure how to go on from here
What will the interval of convergence be in this case?
 
9:49 AM
@robjohn are you there ?
 
 
1 hour later…
11:01 AM
@Paradox101 Why not try the ratio test?
 
user174558
11:13 AM
I am now an orange.
 
11:57 AM
@TobiasKildetoft Your suspicion that it fails in the infinite case is verified here: math.stackexchange.com/a/1306293/284674
@JasperLoy That's a really nice color
 
user174558
@robjohn We are now of the same colour.
 
user174558
@I'mmostlyjustanidiot Thanks no need to call yourself idiot.
 
@JasperLoy Only mostly an idiot, not fully
 
@JasperLoy what happened to your blue?
 
@I'mmostlyjustanidiot Neat
 
user174558
11:58 AM
@robjohn Well, I thought I would have a change, at least for a while.
 
user174558
It's troublesome that there are so many definitions of the Fourier transform.
 
@JasperLoy IMO, the one that Fourier Analysts should use is $$\int_{-\infty}^\infty f(x)e^{-2\pi ix\xi}\mathrm{d}x$$
 
user174558
@robjohn I see. Rudin uses another one though. Does it mean he is not a mathematician?
 
@JasperLoy Does he use the one with $e^{-ix\xi}$?
 
user174558
@robjohn Yes, same as Michael Taylor and Lawrence Evans.
 
12:03 PM
@JasperLoy That is the one that physicists usually use since it is easier to write. However, the inverse transform is different and other properties are not as nice.
 
It's weird that the chat still does not support Mathjax
 
user174558
@robjohn That is the one Stein uses, and also Gerald Folland.
 
@JasperLoy Folland was a Stein student, if I am not mistaken
 
user174558
@robjohn I see. I think I will use what you suggested then. That also seems to be the one most Fourier analysts use.
 
user174558
@robjohn Stein must be your student then!
 
12:04 PM
@JasperLoy Perhaps I should modify my claim, then...
 
@AntonioVargas how will a simple ratio test help? I have to use uniform convergence by the M test as well. Although for the M series I used the ratio test for values of $r$
 
user174558
@robjohn I am trying to like the Stein books but they just don't seem to contain enough material. The 4 Lectures in Analysis books.
 
Is that what you meant or should I apply the ratio test from the start? @AntonioVargas
 
@JasperLoy Those were written much too late. I never read them.
 
user174558
@robjohn Of course, the other 3 main ones are great. Anyway, most top algebraists seem to define ring to have 1, so I will also follow that convention.
 
12:09 PM
@Aldon It does, you just have to get the link off of the thing on the right
 
user174558
Wow, this orange is really beautiful. Maybe I should start eating oranges now and not bananas.
 
@Paradox101 $$\begin{align}\binom{-\frac12}{n} &=\frac{\left(-\frac12\right) \left(-\frac32\right) \left(-\frac52\right)\cdots \left(-\frac{2n-1}2\right)}{n!}\\ &=\frac{(-1)^n}{2^nn!}(2n-1)!!\\ &=\frac{(-1)^n}{2^nn!}\frac{(2n)!}{2^nn!}\\ &=\left(-\frac14\right)^n\binom{2n}{n}\end{align}$$
 
That orange makes me happy
 
user174558
@robjohn Today is Thanksgiving Day. I am thankful for my mum and my books, but nothing else.
 
user174558
@I'mmostlyjustanidiot The sun rises just before it becomes darkest.
 
12:11 PM
@JasperLoy How?
 
user174558
@I'mmostlyjustanidiot Just a saying.
 
I don't like it
 
user174558
@I'mmostlyjustanidiot It means something like don't give up when you feel like.
 
user174558
Did you know that Book Depository was started by a former Amazon employee and is now acquired by Amazon? Interesting.
 
Jasper, are you a math student?
Would you like to discuss Noetherian rings?
 
user174558
12:14 PM
I am a mental patient trying to get well, not a math student, sorry.
 
I am a math student trying to get mental
 
user174558
You can discuss math with the experts here, I am only an orange.
 
@JasperLoy But you're the apple of my eye.
 
I'm mostly just an idiot
 
user174558
@ChantryCargill Hehe. Your name is interesting. It sounds Indian.
 
12:16 PM
@JasperLoy It's from a cowboy book I believe.
 
user174558
@ChantryCargill So it is not your real name? I am using mine here.
 
@JasperLoy It is my real name.
It was based off of a series by Louis L'amour. I think it was borden chantry.
 
user174558
@ChantryCargill Ah? Your parents like cowboys then.
 
user174558
Interesting that cowboys ride on horses and not cows. They should be called horseboys.
 
@JasperLoy Interesting observation.
 
user174558
12:18 PM
@ChantryCargill I read that you want to go to grad school and do math. What are you interested in?
 
user174558
Oh my mum is back, time for dinner, bye.
 
lol
He must be hungry.
 
@robjohn ok I got i. Thanks :)
 
hello @robjohn
 
@robjohn can I ask you a question relate to analysis?
 
12:27 PM
@JasperLoy The explanation is easy: A cowboy is the boy who herds the cows.
 
12:43 PM
@JasperLoy is that a closed-eye mean square that u had in avatar, i mistook u for rob from far
 
@Agawa001 It's actually the back of his head.
 
lol :D
 
1:05 PM
@Paradox101 I missed the part where you have to use the M-test.
 
@evinda try this v = symsum(4/(k^2*pi^2)*(sin(k*pi/2)*sin(k*pi*x))'*exp(-k^2*pi^2*t), k, 1, Inf)
 
@AntonioVargas then is what I've done correc? If so how do I poced from thee?
 
1:20 PM
@Paradox101 I'm not sure what you have done. It wasn't clear from the image.
 
1:41 PM
@AntonioVargas this is what I've done: $ \sum \frac{1}{\sqrt n}(\frac{x}{1+x})^n \le \sum \frac{1}{\sqrt n}x^n=\sum \frac{1}{\sqrt n}r^n= M_n$ from ratio test $r<1$ $\implies$ $-r<(\frac{x}{1+x})^n<r$
 
@Paradox101 Probably the first step is not correct (unless you also assume $x>0$?). You'd be better off setting $x/(1+x) = y$ and finding the radius of convergence for $y$ in $\sum n^{-1/2} y^n$ first, then substitute back in $y = x/(1+x)$ to find the interval of admissible $x$'s.
 
How does achille swap out n for x d/dx here? math.stackexchange.com/a/1547255/109879
 
Huy
does Heine Borel imply locally compact?
 
1:56 PM
@AntonioVargas if I do that, then wouldn't it end in the same manner? $ \sum \frac{1}{\sqrt n}(\frac{x}{1+x})^n \le \sum \frac{1}{\sqrt n}r^n=\sum \frac{1}{\sqrt n}r^n= M_n$ from ratio test $r<1$ $\implies$ $-r<(\frac{x}{1+x})^n<r$
 
@Paradox101 Sure, except now you've taken a valid path there :)
 
@AntonioVargas so then in the final answer we'll simply replace $r$ with $1$?
 
2:20 PM
@Paradox101 sorry, I was away.
 
@robjohn no problem
 
@JasperLoy No, I was a Stein student.
@Paradox101 did you get your question answered?
 
@robjohn yes almost. Given the question regarding the M-test in the picture is the last point in this solution $ \sum \frac{1}{\sqrt n}(\frac{x}{1+x})^n \le \sum \frac{1}{\sqrt n}^n=\sum \frac{1}{\sqrt n}r^n= M_n$ from ratio test $r<1$ $\implies$ $-r<(\frac{x}{1+x})^n<r \implies |(\frac{x}{1+x})^n|< 1$ <$ the answer?
 
2:36 PM
what picture?
 
@robjohn this one
 
@Paradox101 You need $\left|\frac{x}{1+x}\right|\lt1$
 
@robjohn oh ok but after we remove $n$ from there that will simply be the answer we need?
 
@Paradox101 Not sure what you mean by "remove $n$ from there"
@Paradox101 To get uniform convergence, you need $\left|\frac{x}{1+x}\right|\le r\lt1$
Have to go for a while.
 
2:51 PM
How do I sketch the support for a probability function p_{X,Y}: supp p_{X,Y}={(x,y)|x=-1,0,1,y=1,2,...} ?
I can't imagine how it looks!
If anyone could lead me on the way or draw a crude sketch, it would be immensely helpful
 
3:06 PM
@robjohn I mean that as we've assigned $r$ the value $(\frac{x}{1+x})$ then there will be no $n$ here. So will this simply be the answer we were looking for?
ok
 
3:41 PM
@PVAL I looked through some of the transcript to see if it got answered but apparently not enough
 
I've reformulated my troubles with discrete, stochastic vectors in a example question: math.stackexchange.com/questions/1547402/…
Feel free to help
I have enumerated the things I don't know how to work out
This is for my exam prep
 
3:58 PM
Hey guys, just a question.
So, I have that, if X is a complex inner product space and T a linear endomorphism of X, then, if <Tx,x> = 0 for all x in X, we must have T = 0.
Thing is, consider the space spanned by all finite linear combinations of {e^(imx), m \in \mathbb{N}}
That's a complex inner product space given by the inner product, <f,g> = f \bar{g}.
 
@AlpArslan why does <Tx,x>=0 for all x imply T=0?
also $f(x)\overline{g(x)}$ is a function, not a scalar. you mean $\int_{-\pi}^\pi f(x)\overline{g(x)}\,{\rm d}x$
 
My apologies, I left out the integral.
Yes, that's right.
Ach, no, I found the error in the example.
I was about to say, let T: e^(imx) \to e^(i(m+1)x)
But, yes, yes, I see why it doesn't work.
Hmm, besides just screwing with the algebra, is there a neater way of proving that <Tx,x> = 0 implies T = 0?
 
well, it's not true for real inner product spaces
nor for bilinear forms on complex spaces (the complex inner product is sesquilinear not bilinear)
 
Yes, that's right.
Hmm.
So, the key lies in the sesquilinearity, ofcourse.
But, ach, I'm getting lost in circular algebra
 
4:20 PM
Guys, recently I've been thinking that my problem solving skills seem to be quite poor - do you think there would be some use in solving things like math olympiads etc, despite the fact that I'm studying at a university?
I think sort of missed that part in my life - while other good students were solving problems of that kind, I was busy playing video games xD
 
Hi!!!
Is $\langle x, \langle x,y \rangle y \rangle $ equal to $\langle x,y \rangle^2$ ?
 
if $\lambda$ is a scalar, $\langle x,\lambda y\rangle$ is equal to $\overline{\lambda}\langle x,y\rangle$ if you're a mathematician and equal to $\lambda\langle x,y\rangle$ if you're a physicist
 
I study applied mathematics... and we have seen the property $\langle x,\lambda y\rangle=\lambda\langle x,y\rangle$. @anon
 
are you using real numbers or complex numbers?
 
Real numbers... aa in this case $\overline{\lambda}=\lambda$, right? @anon
 
4:31 PM
yes
 
A ok... Let $X$ be a Hilbert space ( with inner product $\langle, \rangle $).
The following inequality holds:

$$(\star) |\langle x,y \rangle| \leq ||x|| \cdot ||y|| , \forall x,y \in X$$

@anon
@anon I have also an other question
The inequality $(\star)$ is equivalently written as $\langle x,y \rangle^2 \leq \langle x,x \rangle \langle y,y \rangle , x,y \in X (\star \star)$.

If $y=0$ then $(\star \star)$ holds since $\langle x,0 \rangle =0$ and $\langle 0,0 \rangle=0$.

It suffices to show that $(\star \star)$ holds for $x \in X$ and $y \neq 0$ such that $\langle y,y \rangle=1$.

(if $y \in X \setminus{ \{0\} }$ and $||y|| \neq 1$ then we pick as $y$ the quantity $\frac{y}{||y||}$)


Let $y \in X \setminus{ \{ 0 \} }$ with $\langle y,y \rangle=1$ and $x \in X$.
 
sure you do
if you scale $y$ in $(\star\star)$ you still get a true equality
Claim I: `(**)` holds for all x,y
Claim II: `(**)` holds for all x and unit-norm y
Can you prove Claim I implies Claim II? Can you prove Claim II implies Claim I?
 
4:51 PM
@anon We suppose that it holds that $\langle x,y \rangle ^2 \leq \langle x,x \rangle \langle y,y \rangle$.

If $||y||=1 \Rightarrow ||y||^2=1 \Rightarrow \langle y,y \rangle=1$.
So we get that $\langle x,y \rangle ^2 \leq \langle x,x \rangle$.

Now, let's suppose that $\langle x,y \rangle^2 \leq \langle x,x \rangle$ for $y \in X$ with $||y||=1$.
We pick as y , $\frac{y}{||y||}$ and we get: $ \langle x, \frac{y}{||y||} \rangle^2 \leq \langle x,x \rangle \Rightarrow \frac{1}{||y||^2} \langle x,y \rangle^2 \leq \langle x,x \rangle \Rightarrow \langle x,y \rangle^2 \leq \langle x,x \rangle \lan
@anon Is it right like that?
 
more or less
 
Could I improve something?@anon
 
your first sentence should say "for all y." you don't need to write any equations down to explain why Claim I implies Claim II. It actually doesn't make sense to say "we pick as y, y/||y||," you have to find a better way of saying that. In fact just your equation is fine for proving Claim II implies Claim I.
 
Ok.. Thank you!!! @anon
 
00:00 - 17:0017:00 - 00:00

« first day (1940 days earlier)      last day (3082 days later) »